Difference between revisions of "2024 AMC 8 Problems/Problem 19"

(Solution)
m (Solution 1)
Line 10: Line 10:
 
==Solution 1==
 
==Solution 1==
  
We first
+
We first start by finding the amount of red and white sneakers. /dfrac{3}{5}*15
  
 
==Video Solution by Math-X (First fully understand the problem!!!)==
 
==Video Solution by Math-X (First fully understand the problem!!!)==

Revision as of 18:41, 25 January 2024

Problem

Jordan owns $15$ pairs of sneakers. Three fifths of the pairs are red and the rest are white. Two thirds of the pairs are high-top and the rest are low-top. The red high-top sneakers make up a fraction of the collection. What is the least possible value of this fraction?

(A) $0$ (B) $\frac{1}{5}$ (C) $\frac{4}{15}$ (D) $\frac{1}{3}$ (E) $\frac{2}{5}$

Solution

Jordan has $10$ high top sneakers, and $6$ white sneakers. We would want as many white high-top sneakers as possible, so we set $6$ high-top sneakers to be white. Then, we have $10-6=4$ red high-top sneakers, so the answer is $\boxed{\dfrac{4}{15}}.$

Solution 1

We first start by finding the amount of red and white sneakers. /dfrac{3}{5}*15

Video Solution by Math-X (First fully understand the problem!!!)

https://www.youtube.com/watch?v=eYnLh_SGy7c

~Math-X